Puzzle practice test

1.

Santosh was travelling in a railway compartment. There he met a man and five women. Each woman had a child in her arms. How many were there in the compartment?

Answer: Option D
2.

SELECTION BASED ON GIVEN CONDITIONS

A few essential criteria for selection of group of items are given: From amongst five doctors A,B,C,D and E;four engineers G,H,K and L; six teachers M,N,O,P,Q and R some teams are to be selected.A,B,G,H,O,P and Q are females and the rest are males.
Where ever there is a male doctor ,no female teacher.
Where ever there is a male engineer, no female doctor.
There shall not be more than two male teachers in any team.
If the team consists of 2 doctors, 3 female teachers and 2 engineers, the members of the team are:
Answer: Option A

Explanation:

Doctors are A B C D E

Female teachers are O P Q

Engineers are G H K L

Male doctor and no female teacher => A,B are Doctors.

So the Team consists of A B O P Q G H.

3.

If the team consists of 2 doctors,1 engineer and 4 teachers all the following are possible except

Answer: Option D

Explanation:

Teachers are M N O P Q R

Four are needed
Three are male teachers.
Female teachers are also to be selected.Hence no male doctors C D E are selected.
So Doctors => A B
Both doctors are Females =>no male Engineer to be selected.
Ans:Team =>A B K N R P Q.
4.

If the team consists of 2 doctors, 2 female teachers and two engineers all the following teams are possible except:

Answer: Option C

Explanation:

The doctors are A B C D E,

Female Teachers are O P Q,
Engineers are G H K L,
Since 2 Female
Teachers are to be selected.
So male doctors i.e;C D E cannot be selected. So, 2 Doctors selected will be A,B. Both Doctors are Females. So, male Engineer K L cannot be selected. G H are chosen. =>A B K L P Q are selected.
5.

If the team consists of 2 doctors, 3 female teachers and 2 engineers, the members of the team are:

Answer: Option A

Explanation:

Doctors are A B C D E
Female teachers are O P Q
Engineers are G H K L
Male doctor and no female teacher => A,B are Doctors.
So the Team consists of A B O P Q G H.

6.

Directions (Q5-9): Read the following information carefully and answer the questions given below it:-

K,L,M,N,O,P,Q,R,S,U and W are only ten members in a department. There is a proposal to form a team from with in the members of the department, subject to the following conditions.

· A team must include exactly one among P,Q and S· A team must include either M or Q , but not both· If a team includes K, then it must also include L, and vice versa.· If a team includes on among S,U and W, then it must also include the other two· L and N cannot be members of the same team· L and U cannot be members of the same team?

who cannot be member of a team size 3 ?
Answer: Option D

Explanation:

If L is in the team K will also be in the same Team. Exactly one among the P,R and S and exactly one among the M and Q are included in the team. hence when L is in the team , the team strength will be atleast four. Hence L cannot be in the team of three.

Options (2) ,(3) and (4) : M,P and N is a Possible TeamOptions (5) : P,Q and N is a possible team.

7.

who can be a member of a team size 5 ?

Answer: Option C

Explanation:

Options (1) and (2) : If K is in the team , then L is laso included and vice versa.

ð Neither U nor N is included.When U is not included neither S nor W is included.From (ii) , only one among M and Q is included.From (i) Only one among P and R is included.Hence , the team can be K L M P or K L M R or K L Q P or K L Q ROption (3) :If M is included , then q cannot be included. If S is included then both U and W are included.Hence , the team is M , S , U , W and N.Option (4) and (5) : If any one of P or R is included then S cannot be included.ð U and W are also not included.Only one among M and Q is included.If K is included , then L is also included.ð N cannot be included.ð Hence , the team can be (P,M or Q) , (K,L or R) ( K,L and M or Q)

8.

what would be the size of a team includes K ?

Answer: Option C

Explanation:

If K is included , then L is also included.

ð Both N and U are not included.Since U is not included neither of s and W is included.ð The term consists of only four members

9.

what would be the size of the largest possible team ?

Answer: Option B

Explanation:

To have the largest possible team we should include S , U and W.

Neither P nor R is included.Only one among M and Q is included.Both K and L can included.Since , L is included neither N nor U can be included.Hence , the maximum possible strength of the team is 5.

10.

In how many ways a team can be constituted so that the team includes N ?

Answer: Option C

Explanation:

If N is included , then L is not included.

ð K is not included.But U may or may not be included.Case (i): U is included.ð S and W are also included.ð Neither P nor R is included.One among N nor R is included.Hence , the possible teams are N , U , S , W and P or N , U , S , W and Q.

Leave a Reply0

Your email address will not be published.